8 svar
43 visningar
destiny99 är nöjd med hjälpen
destiny99 Online 6978
Postad: 4 jan 2023 10:27

Konvergens/divergens serier

Hej!

 

 

Jag fastnade på denna fråga 5a. Hur ska man först börja? De ger oss att k=1 men inget mer än så. 

SeriousCephalopod 2692
Postad: 4 jan 2023 10:44 Redigerad: 4 jan 2023 10:48

När man ska beräkna ett gräsvärde av en summa av positiva termer så finns det fyra kvalitativt skilda möjligheter

1. Konvergerar mot 0

2. Konvergerar inte (dvs går mot oändligheten)

3. Konvergerar mot ett reellt tal som kan uttryckas med ett uttryck med elementära konstanter som π2e/5\pi^2 e / 5

4. Konvergerar mot ett reellt tal som inte kan uttryckas med ett uttryck med elementära konstanter

På en tenta kan man exkludera 4 på rak arm då det inte är ett svar som kan ges vid en tenta. 3 är möjligt men jag skulle klassa det som osannoligt då serien liknar p-serien k\sum \sqrt{k} och det är välkänt att serier på formen kp\sum k^{p} inte har elementära formler förutom i ett begränsat antal fall som p = -2, och fallet p = 1/2 är inte en av dem

Därför skulle jag på rak arm anta att gränsvärdet antingen inte finns eller är 0 (alt 1 eller 2) och istället för att försöka hitta ett ett rellt tal, 0 < x < infty, logiskt bevisa att det är något av de fallen.

tillägg: Min diskussion handlar om k\sqrt{k}-termerna då de summan av de vänstra termerna bara är 2.

k=1n2/n=n(2/n)=2\sum_{k = 1}^n 2/n = n(2/n) = 2

så svårigheterna ligger bara i summan av den högra termen.

(heuristiken behöver såklart inte stämma då det är en heuristik och inte ett bevis i sig men poängen här är att man måste börja med att ustesluta möjligheter med en gissning om vad som är rimligt för annars kan man råka försöka lösa problem som inte går att lösa)

destiny99 Online 6978
Postad: 4 jan 2023 11:08 Redigerad: 4 jan 2023 11:13
SeriousCephalopod skrev:

När man ska beräkna ett gräsvärde av en summa av positiva termer så finns det fyra kvalitativt skilda möjligheter

1. Konvergerar mot 0

2. Konvergerar inte (dvs går mot oändligheten)

3. Konvergerar mot ett reellt tal som kan uttryckas med ett uttryck med elementära konstanter som π2e/5\pi^2 e / 5

4. Konvergerar mot ett reellt tal som inte kan uttryckas med ett uttryck med elementära konstanter

På en tenta kan man exkludera 4 på rak arm då det inte är ett svar som kan ges vid en tenta. 3 är möjligt men jag skulle klassa det som osannoligt då serien liknar p-serien k\sum \sqrt{k} och det är välkänt att serier på formen kp\sum k^{p} inte har elementära formler förutom i ett begränsat antal fall som p = -2, och fallet p = 1/2 är inte en av dem

Därför skulle jag på rak arm anta att gränsvärdet antingen inte finns eller är 0 (alt 1 eller 2) och istället för att försöka hitta ett ett rellt tal, 0 < x < infty, logiskt bevisa att det är något av de fallen.

tillägg: Min diskussion handlar om k\sqrt{k}-termerna då de summan av de vänstra termerna bara är 2.

k=1n2/n=n(2/n)=2\sum_{k = 1}^n 2/n = n(2/n) = 2

så svårigheterna ligger bara i summan av den högra termen.

(heuristiken behöver såklart inte stämma då det är en heuristik och inte ett bevis i sig men poängen här är att man måste börja med att ustesluta möjligheter med en gissning om vad som är rimligt för annars kan man råka försöka lösa problem som inte går att lösa)

Så hur ska jag börja ? Jag kan knappt lösa uppgiften och ser inga mönster eller så.

 

Men jag fick en liten tanke helt plötsligt. Om vi bryter ut 2/n så har vi 2/n( 1+sqrt(2*1/n)). Då kan vi typ resonerar som man gör med gränsvärde dvs vi har vad händer med nämnaren när n går mot oändligheten. Då  kommer kvoten gå mot noll så vi har inom parentesen 1+0 medan 2/oändlighet som vi bröt ut förut kommer också gå mot 0 då n=> oändlighet. Så hela serien konvergerar mot 0. Alltså är gränsvärdet 0

SeriousCephalopod 2692
Postad: 4 jan 2023 11:13 Redigerad: 4 jan 2023 11:15

limnk=12n+2n2kn=2+limnk=1n2n2kn\lim_{n \to \infty} \sum_{k =1}^\infty \left ( \frac{2}{n} + \frac{2}{n}\sqrt{\frac{2k}{n}}\right ) = 2 + \lim_{n \to \infty} \sum_{k = 1}^n \frac{2}{n}\sqrt{\frac{2k}{n}}

Visa att gränsvärdet för summan till höger antingen går mot 0 eller går mot oändligheten med hjälp av jämförelsesatser.

destiny99 Online 6978
Postad: 4 jan 2023 11:15 Redigerad: 4 jan 2023 11:15
SeriousCephalopod skrev:

limnk=1n2+2n2kn=2+limnk=1n2n2kn\lim_{n \to \infty} \sum_{k =1}^\infty \left ( \frac{n}{2} + \frac{2}{n}\sqrt{\frac{2k}{n}}\right ) = 2 + \lim_{n \to \infty} \sum_{k = 1}^n \frac{2}{n}\sqrt{\frac{2k}{n}}

Visa att gränsvärdet för summan till höger antingen går mot 0 eller går mot oändligheten med hjälp av jämförelsesatser.

Jag löste på annat sätt istället och fick gränsvärde är 0 utan jämförelse sats eller något annat.  Jag behandlade frågan som en vanlig gränsvärde fråga.

SeriousCephalopod 2692
Postad: 4 jan 2023 11:19

Men jag fick en liten tanke helt plötsligt. Om vi bryter ut 2/n så har vi 2/n( 1+sqrt(2*1/n)). Då kan vi typ resonerar som man gör med gränsvärde dvs vi har vad händer med nämnaren när n går mot oändligheten. Då  kommer kvoten gå mot noll så vi har inom parentesen 1+0 medan 2/oändlighet som vi bröt ut förut kommer också gå mot 0 då n=> oändlighet. Så hela serien konvergerar mot 0. Alltså är gränsvärdet 0

Nej det stämmer inte. Gränsvändets värde är med säkerhet minst 2.

Bara eftersom en summas termer går mot 0 så måste inte summans värde går mot noll eftersom samtidigt som termernas storlekar minskar så ökar antalet termer man adderar

Exempel:

k=1n1n=1n+1n+...+1n=1\sum_{k = 1}^n \frac{1}{n} = \frac{1}{n} + \frac{1}{n} + ... + \frac{1}{n} = 1

oavsett hur stort n.

Kvalitativt: En summa av ett oändligt antal små saker kan fortfarande vara större än 0.

destiny99 Online 6978
Postad: 4 jan 2023 11:21 Redigerad: 4 jan 2023 11:25
SeriousCephalopod skrev:

Men jag fick en liten tanke helt plötsligt. Om vi bryter ut 2/n så har vi 2/n( 1+sqrt(2*1/n)). Då kan vi typ resonerar som man gör med gränsvärde dvs vi har vad händer med nämnaren när n går mot oändligheten. Då  kommer kvoten gå mot noll så vi har inom parentesen 1+0 medan 2/oändlighet som vi bröt ut förut kommer också gå mot 0 då n=> oändlighet. Så hela serien konvergerar mot 0. Alltså är gränsvärdet 0

Nej det stämmer inte. Gränsvändets värde är med säkerhet minst 2.

Bara eftersom en summas termer går mot 0 så måste inte summans värde går mot noll eftersom samtidigt som termernas storlekar minskar så ökar antalet termer man adderar

Exempel:

k=1n1n=1n+1n+...+1n=1\sum_{k = 1}^n \frac{1}{n} = \frac{1}{n} + \frac{1}{n} + ... + \frac{1}{n} = 1

oavsett hur stort n.

Kvalitativt: En summa av ett oändligt antal små saker kan fortfarande vara större än 0.

Hm okej men då kan jag ingenting om det här området som har just med serier att göra. Låter lite krångligt att lösa den uppgiften då. Får kolla upp och sen lösa om den sen.   Tror frågan i sig är ej krångligt och sen behöver man behandla såna frågor annorlunda än om det bara hade stått avgör bla bla. Det är 2 olika situationer och för att ej krångla till för sig själv behöver man fokusera på vad frågan vill att du ska göra. Tyvärr är det ej lätt för mig att se vad jag ska göra om någon annan bara ser med ögonblicket. Tack ändå.

SeriousCephalopod 2692
Postad: 4 jan 2023 11:29 Redigerad: 4 jan 2023 11:30

Principen här brukar kallas 'jämföra summor' och bör ligga under något avsnitt med de nyckelorden.

Principen som jag förväntar mig är att om aka_k och bkb_k är två talföljder med positiva termer sådana att ak<>a_k <> då gäller det att ak<bk\sum a_k < \sum=""> vilket man kan använda för den aktuella delsumman genom att välja ett bkb_k så att summan bk\sum b_k blir blir möjlig att bestämma exakt även när ak\sum a_k inte går att bestämma exakt.

destiny99 Online 6978
Postad: 4 jan 2023 11:37 Redigerad: 4 jan 2023 11:38
SeriousCephalopod skrev:

Principen här brukar kallas 'jämföra summor' och bör ligga under något avsnitt med de nyckelorden.

Principen som jag förväntar mig är att om aka_k och bkb_k är två talföljder med positiva termer sådana att ak<>a_k <> då gäller det att ak<="">\sum a_k < \sum=""> vilket man kan använda för den aktuella delsumman genom att välja ett bkb_k så att summan bk\sum b_k blir blir möjlig att bestämma exakt även när ak\sum a_k inte går att bestämma exakt.

Jag har hört om det men jag ser ej hur jämförelse är aktuell i denna fråga. Låter bara oklart för mig när frågan säger beräkna gränsvärdet. Men kollar upp sånt snart som sagt. 

Svara Avbryt
Close